Help
I have no idea why the answer goes to (A). Please explain this. Thanks,
Batman on May 21, 2015
  • June 1993 LSAT
  • SEC3
  • Q23
2
Replies
Help
I chose (E) by eliminating less correct answers, not by knowing why (E) must be the answer. Could...
Batman on May 21, 2015
  • February 1993 LSAT
  • SEC3
  • Q13
2
Replies
Explanation needed
Can someone please explain the correct answer here? Thanks!
Tester on May 18, 2015
  • June 2014 LSAT
  • SEC1
  • Q1
1
Reply
Help
Could you please explain why (B) is the answer? I thought (b) could logically begin immediately a...
Batman on May 17, 2015
  • June 1991 LSAT
  • SEC1
  • Q27
2
Replies
Setup and deductions
Can someone please explain the setup of this game, along with any initial deductions? Thank you.
iup8man on May 14, 2015
  • June 2000 LSAT
  • SEC1
  • Q8
3
Replies
Help
I eliminated choice (c), (d),and (e) on the basis of 4th paragraph. I narrowed down to both (a) a...
Batman on May 9, 2015
  • December 1994 LSAT
  • SEC3
  • Q3
2
Replies
Help,
I need help to understand this question
Lamontenay on May 5, 2015
  • June 2007 LSAT
  • SEC1
  • Q8
1
Reply
Question5
Hello, Would anyone be able to help with question if; if Freedom were to make a voyage to T in We...
Roxibrahim on April 30, 2015
  • June 2007 LSAT
  • SEC1
  • Q13
2
Replies
Help
I don't understand why the answer is (B), not (A). Please, explain this. Thanks,
Batman on April 29, 2015
  • December 1995 LSAT
  • SEC4
  • Q16
2
Replies
Error
I believe there's an error in the sequencing game questions the first game seems to have the wron...
Montero on April 28, 2015
  • February 1992 LSAT
  • SEC3
  • Q4
1
Reply
Help
I'm struggling to understand what this question really means. Is the answer (a), in fact, either ...
Batman on April 22, 2015
  • September 1995 LSAT
  • SEC4
  • Q10
2
Replies
Help
Hey, it's been a while!!! 😉I'm still wondering why (b) is not the answer Doesn't "a jury may g...
Batman on April 17, 2015
  • December 1994 LSAT
  • SEC3
  • Q27
4
Replies
Clarification
Could you compare answer choices A and E?
tselimovic on March 30, 2015
  • June 2011 LSAT
  • SEC1
  • Q24
1
Reply
Necessary Assumption
Could you explain the negation technique to locate the correct answer for necessary-assumption qu...
Spring on March 28, 2015
  • December 2011 LSAT
  • SEC1
  • Q21
1
Reply
Flawed Parallel Reasoning
Please explain this question. Thanks,
Spring on March 27, 2015
  • December 2011 LSAT
  • SEC1
  • Q11
1
Reply
Paradox question
Please explain this question. Thanks,
Spring on March 25, 2015
  • December 2011 LSAT
  • SEC1
  • Q12
1
Reply
Clarification
Could you please explain what makes (E) correct?
tselimovic on February 19, 2015
  • December 2012 LSAT
  • SEC2
  • Q14
1
Reply
Clarification
Could you please explain what makes (B) correct?
tselimovic on February 11, 2015
  • December 2012 LSAT
  • SEC2
  • Q13
1
Reply
Diagramming this is confusing
Not sure how to diagram this answer choice! I realized quickly that it wasn't Answer Choices C,D,...
Per5ection on February 10, 2015
  • December 1996 LSAT
  • SEC2
  • Q21
1
Reply
Help
Here is what I set up logical structures in the stimulus: "(If or when)The govt's restriction on ...
Batman on February 7, 2015
  • December 2002 LSAT
  • SEC2
  • Q12
5
Replies